Bạn chưa đăng nhập. Vui lòng đăng nhập để hỏi bài

Những câu hỏi liên quan
KCLH Kedokatoji
Xem chi tiết
chu van anh
Xem chi tiết
tran hoang dang
14 tháng 1 2017 lúc 20:41

minh ko biet xin loi ban nha

minh ko biet xin loi ban nha

minh ko biet xin loi ban nha

minh ko biet xin loi ban nha

Trần Quốc Đạt
15 tháng 1 2017 lúc 12:25

\(a_3=3,a_4=\frac{11}{3}\) nên đề sai rồi nha bạn.

Tăng Văn Minh
15 tháng 1 2017 lúc 15:31

\(\hept{\begin{cases}a_1=a_2=1\\a_n=\frac{a_{n-1}^2+2}{a_{n-2}}\end{cases}}\) như vậy ms đúng đề bạn ơi

loancute
Xem chi tiết
3536828
Xem chi tiết
Trần Quốc Tuấn hi
Xem chi tiết
NGUYỄN MINH HUY
Xem chi tiết
Nguyễn Việt Lâm
8 tháng 3 2021 lúc 5:17

a.

\(\Leftrightarrow na_{n+2}-na_{n+1}=2\left(n+1\right)a_{n+1}-2\left(n+1\right)a_n\)

\(\Leftrightarrow\dfrac{a_{n+2}-a_{n+1}}{n+1}=2.\dfrac{a_{n+1}-a_n}{n}\)

Đặt \(b_n=\dfrac{a_{n+1}-a_n}{n}\Rightarrow\left\{{}\begin{matrix}b_1=\dfrac{a_2-a_1}{1}=1\\b_{n+1}=2b_n\end{matrix}\right.\) \(\Rightarrow b_n=2^{n-1}\Rightarrow a_{n+1}-a_n=n.2^{n-1}\)

\(\Leftrightarrow a_{n+1}-\left[\dfrac{1}{2}\left(n+1\right)-1\right]2^{n+1}=a_n-\left[\dfrac{1}{2}n-1\right]2^n\)

Đặt \(c_n=a_n-\left[\dfrac{1}{2}n-1\right]2^n\Rightarrow\left\{{}\begin{matrix}c_1=a_1-\left[\dfrac{1}{2}-1\right]2^1=2\\c_{n+1}=c_n=...=c_1=2\end{matrix}\right.\)

\(\Rightarrow a_n=\left[\dfrac{1}{2}n-1\right]2^n+2=\left(n-2\right)2^{n-1}+2\)

Nguyễn Việt Lâm
8 tháng 3 2021 lúc 5:21

b.

Câu b này đề sai

Với \(n=1\Rightarrow\sqrt{a_1-1}=0< \dfrac{1\left(1+1\right)}{2}\)

Với \(n=2\Rightarrow\sqrt{a_1-1}+\sqrt{a_2-1}=0+1< \dfrac{2\left(2+1\right)}{2}\)

Có lẽ đề đúng phải là: \(\sqrt{a_1-1}+\sqrt{a_2-1}+...+\sqrt{a_n-1}\ge\dfrac{n\left(n-1\right)}{2}\)

Ta sẽ chứng minh: \(\sqrt{a_n-1}\ge n-1\) ; \(\forall n\in Z^+\)

Hay: \(\sqrt{\left(n-2\right)2^{n-1}+1}\ge n-1\)

\(\Leftrightarrow\left(n-2\right)2^{n-1}+2n\ge n^2\)

- Với \(n=1\Rightarrow-1+2\ge1^2\) (đúng)

- Với \(n=2\Rightarrow0+4\ge2^2\) (đúng)

- Giả sử BĐT đúng với \(n=k\ge2\) hay \(\left(k-2\right)2^{k-1}+2k\ge k^2\)

Ta cần chứng minh: \(\left(k-1\right)2^k+2\left(k+1\right)\ge\left(k+1\right)^2\)

\(\Leftrightarrow\left(k-1\right)2^k+1\ge k^2\)

Thật vậy: \(\left(k-1\right)2^k+1=2\left(k-2\right)2^{k-1}+2^k+1\ge2k^2-4k+2^k+1\)

\(\ge2k^2-4k+5=k^2+\left(k-2\right)^2+1>k^2\) (đpcm)

Do đó:

\(\sqrt{a_1-1}+\sqrt{a_2-1}+...+\sqrt{a_n-1}>0+1+...+n-1=\dfrac{n\left(n-1\right)}{2}\)

Nguyễn Việt Lâm
9 tháng 3 2021 lúc 1:19

c.

Ta có:

\(\dfrac{a_n}{3^n}=\dfrac{\left(n-2\right)2^{n-1}+2}{3^n}=\dfrac{n}{2\left(\dfrac{3}{2}\right)^n}-\left(\dfrac{2}{3}\right)^n+\dfrac{2}{3^n}\)

Đặt \(S_n=\sum\limits^n_{i=1}\dfrac{a_n}{3^n}=\dfrac{1}{2}\sum\limits^n_{i=1}\dfrac{n}{\left(\dfrac{3}{2}\right)^n}-\sum\limits^n_{j=1}\left(\dfrac{2}{3}\right)^n+2\sum\limits^n_{k=1}\dfrac{1}{3^n}=\dfrac{1}{2}S'-2+2\left(\dfrac{2}{3}\right)^n+1-\dfrac{1}{3^n}\)

Xét \(S'=\sum\limits^n_{i=1}\dfrac{n}{\left(\dfrac{3}{2}\right)^n}\)

\(S'=\sum\limits^n_{i=1}\dfrac{n}{\left(\dfrac{3}{2}\right)^n}=\dfrac{1}{\dfrac{3}{2}}+\dfrac{2}{\left(\dfrac{3}{2}\right)^2}+\dfrac{3}{\left(\dfrac{3}{2}\right)^3}+...+\dfrac{n}{\left(\dfrac{3}{2}\right)^n}\)

\(\dfrac{3}{2}S'=1+\dfrac{2}{\dfrac{3}{2}}+\dfrac{3}{\left(\dfrac{3}{2}\right)^2}+...+\dfrac{n}{\left(\dfrac{3}{2}\right)^{n-1}}\)

\(\Rightarrow\dfrac{1}{2}S'=1+\dfrac{1}{\left(\dfrac{3}{2}\right)}+\dfrac{1}{\left(\dfrac{3}{2}\right)^2}+...+\dfrac{1}{\left(\dfrac{3}{2}\right)^{n-1}}-\dfrac{n}{\left(\dfrac{3}{2}\right)^n}=\dfrac{1-\left(\dfrac{2}{3}\right)^n}{1-\dfrac{2}{3}}=3-3\left(\dfrac{2}{3}\right)^n-n\left(\dfrac{2}{3}\right)^n\)

\(\Rightarrow S_n=2-\left(\dfrac{2}{3}\right)^n-\dfrac{1}{3^n}-n\left(\dfrac{2}{3}\right)^n\)

\(\Rightarrow\lim\left(S_n\right)=2\)

Trần Lê Quang Huy
Xem chi tiết
Nguyễn Thị Ngọc Linh
Xem chi tiết
Lê Gia Bảo
8 tháng 8 2017 lúc 20:00

Theo tính chất của dãy tỉ số bằng nha, ta có :

\(\dfrac{a_1}{a_2}=\dfrac{a_2}{a_3}=.....=\dfrac{a_n}{a_{n+1}}=\dfrac{a_1+a_2+....+a_n}{a_2+a_3+....+a_{n+1}}\)

\(\Rightarrow\dfrac{a_1}{a_2}=\dfrac{a_1+a_2+....+a_n}{a_2+a_3+....+a_{n+1}}\)

\(\dfrac{a_2}{a_3}=\dfrac{a_1+a_2+.....+a_n}{a_2+a_3+.....+a_{n+1}}\)

.................................

\(\dfrac{a_n}{a_{n+1}}=\dfrac{a_1+a_2+.....+a_n}{a_2+a_3+.....+a_{n+1}}\)

\(\Rightarrow\left(\dfrac{a_1+a_2+.....+a_n}{a_2+a_3+.....+a_{n+1}}\right)^n=\dfrac{a_1}{a_2}.\dfrac{a_2}{a_3}........\dfrac{a_n}{a_{n+1}}\)

Vậy \(\left(\dfrac{a_1+a_2+......+a_n}{a_2+a_3+......+a_{n+1}}\right)=\dfrac{a_1}{a_{n+1}}\) (đpcm)

~ Học tốt ~

Yuki
Xem chi tiết
Lê Chí Cường
8 tháng 11 2015 lúc 20:54

Đặt \(\frac{a_1}{a_2}=\frac{a_2}{a_3}=...=\frac{a_{n-1}}{a_n}=\frac{a_n}{a_1}=k\)

=>\(\frac{a_1}{a_2}.\frac{a_2}{a_3}.....\frac{a_{n-1}}{a_n}.\frac{a_n}{a_1}=k.k.....k.k\)

=>\(k^n=\frac{a_1.a_2.....a_{n-1}.a_n}{a_2.a_3.....a_n.a_1}\)

=>\(k^n=1=1^n\)

=>k=1

=>\(\frac{a_1}{a_2}=\frac{a_2}{a_3}=...=\frac{a_{n-1}}{a_n}=\frac{a_n}{a_1}=1\)

=>\(a_1=a_2=...=a_n\)

\(=>\frac{a^2_1+a^2_2+...+a_n^2}{\left(a_1+a_2+...+a_n\right)^2}\)

=\(\frac{a^2_1+a^2_1+...+a_1^2}{\left(a_1+a_1+...+a_1\right)^2}\)

=\(\frac{n.a^2_1}{\left(n.a_1\right)^2}=\frac{n.a_1^2}{n^2.a^2_1}=\frac{1}{n}\)

Yuki
8 tháng 11 2015 lúc 21:03

thế này dc ko

Áp dụng t/c của dãy tỉ số bằng nhau, ta có :

\(\frac{a_1}{a_2}=\frac{a_2}{a_3}=...=\frac{a_{n-1}}{a_n}=\frac{a_n}{a_1}=\frac{a_1+a_2+...+a_{n-1}+a_n}{a_2+a_3+...+a_n+a_1}\Rightarrow a_1=a_2=...=a_n\)

\(\frac{a^1_2+a^2_2+...+a^2_n}{\left(a_1+a_2+...+a_n\right)}=\frac{na^2_1}{\left(na_1\right)^2}=\frac{1}{n}\)